Publicaciones Recientes https://www.matetam.com/publicaciones_recientes es P6. La lista de Germán https://www.matetam.com/problemas/combinatoria/p6-lista-german <p>Sea $n$ un entero positivo. Germ&aacute;n tiene una lista de $n$ n&uacute;meros enteros. Si suma todos sus n&uacute;meros, obtiene 6. Si los multiplica, tambi&eacute;n obtiene 6. Encuentra todos los posibles valores para $n$.&nbsp;</p> <fieldset class="fieldgroup group-sugerencia"><legend>Sugerencia</legend><div class="field field-type-text field-field-sugerencia"> <div class="field-label">Sugerencia:&nbsp;</div> <div class="field-items"> <div class="field-item odd"> <p>Observa que si en la lista pertenecen (1, 1, -1, -1), entonces la suma y el producto de los n&uacute;meros de la lista no se ve afectada.</p> </div> </div> </div> </fieldset> <fieldset class="fieldgroup group-sol-sep"><legend>Solución</legend><div class="field field-type-text field-field-sol"> <div class="field-label">Solución:&nbsp;</div> <div class="field-items"> <div class="field-item odd"> <div> Observe que podemos agregar +1-1+1-1 tanto a la suma como al producto y estos permanecer&aacute;n igual. Como estamos agregando 4 elementos, entonces trabajaremos en m&oacute;dulo 4.</div> <div> &nbsp;</div> <div> &nbsp; &nbsp; 6=6. De aqu&iacute; la respuesta es $n=1$. Como $1 \equiv 1 \pmod 4$, entonces un posible valor de $n$ es $n \equiv 1 \pmod 4$ con $n \geq 1$.</div> <div> &nbsp; &nbsp; &nbsp;(-6)(-1)$(1^{13})$=-6-1+13(1). De aqu&iacute; la respuesta es $n=15$. Como $15 \equiv -1 \pmod 4$, entonces un posible valor de $n$ es $n \equiv -1 \pmod 4$ con $n \geq 15$.</div> <div> &nbsp; &nbsp; Observa que el caso (-1)(-1)(6) es analogo al caso 6=6. Asimismo, el caso (-6)(1) es analogo al (-6)(-1).</div> <div> &nbsp; &nbsp; &nbsp;(2)(3)(1)=2+3+1. De aqu&iacute; la respuesta es $n=3$. Como $3 \equiv -1 \pmod 4$, entonces un posible valor de $n$ es $n \equiv -1 \pmod 4$ con $n \geq 3$, y como ya obtuvimos esta respuesta anteriormente, entonces la respuesta 2 esta contenida en este conjunto de soluciones.</div> <div> &nbsp; &nbsp; &nbsp;(-2)(-3)$(1^{11})$=-2-3+11(1). De aqu&iacute; la respuesta es $n=13$. Como $13 \equiv 1 \pmod 4$, entonces un posible valor de $n$ es $n \equiv 1 \pmod 4$ con $n \geq 13$, que est&aacute; contenida dentro de las respuestas del punto 1.</div> <div> &nbsp; &nbsp; &nbsp;(-2)(3)(-1)$(1^6)$=-2+3-1+6(1). De aqu&iacute; la respuesta es $n=9$. Como $9 \equiv 1 \pmod 4$, entonces un posible valor de $n$ es $n \equiv 1 \pmod 4$ con $n \geq 9$ que est&aacute; contenida en las respuestas del punto 1.</div> <div> &nbsp;</div> <div> &nbsp; &nbsp; &nbsp;(-3)(2)(-1)$(1^8)$=-3+2-1+8(1). De aqu&iacute; la respuesta es $n=11$. Como $11 \equiv 1 \pmod 4$, entonces un posible valor de $n$ es $n \equiv -1 \pmod 4$ con $n \geq 11$ que ya esta contenida en el punto 4.</div> <div> &nbsp;</div> <div> Entonces, todas las respuestas son $n\equiv \pm1 \pmod 4 \ \forall \ n \in \mathbb{N}$</div> </div> </div> </div> </fieldset> https://www.matetam.com/problemas/combinatoria/p6-lista-german#comments Combinatoria Números Intermedio Selectivo Final OMM Tamaulipas 2024 Sat, 19 Oct 2024 20:16:47 +0000 Samuel Elias 4133 at https://www.matetam.com P5. Dos circunferencias, una perpendicular. https://www.matetam.com/problemas/geometria/p5-dos-circunferencias-una-perpendicular <p>Sea $ABC$ un tri&aacute;ngulo acut&aacute;ngulo y $\omega$ su circunc&iacute;rculo. Sea $\Gamma$ un c&iacute;rculo con centro $A$ de forma que corta al arco $AB$ que no contiene a $C$ de $\omega$ en un punto $D$ y al arco $AC$ que no contiene a $B$ de $\omega$ en un punto&nbsp; $E$. Sea $K$ la intersecci&oacute;n de $BE$ con $CD$ de tal forma que $K$ est&eacute; sobre $\Gamma$. Demuestra que $AK$ es perpendicular a $BC$.</p> <fieldset class="fieldgroup group-sugerencia"><legend>Sugerencia</legend><div class="field field-type-text field-field-sugerencia"> <div class="field-label">Sugerencia:&nbsp;</div> <div class="field-items"> <div class="field-item odd"> <p>Como $A$ es centro de $\Gamma$, entonces tendremos varios tri&aacute;ngulos is&oacute;sceles. Observa que hay &aacute;ngulos que abren arcos de circunferencias distintas. Utiliza que una bisectriz tambi&eacute;n es altura de un tri&aacute;ngulo is&oacute;sceles.&nbsp;</p> <p>&nbsp;</p> <p>Demuestra que $K$ es el ortocentro de $ABC$</p> </div> </div> </div> </fieldset> <fieldset class="fieldgroup group-sol-sep"><legend>Solución</legend><div class="field field-type-text field-field-sol"> <div class="field-label">Solución:&nbsp;</div> <div class="field-items"> <div class="field-item odd"> <div> Vamos a usar la propiedad de que 1 &aacute;ngulo central es igual al doble de su &aacute;ngulo inscrito que abra el mismo arco.&nbsp;</div> <div> &nbsp;</div> <div> $\angle DAK=2\angle DEK=$ y $\angle DEK=\angle DAB$, y como $AD=AK$, entonces $AB$ es todotriz de $\angle DAK$.&nbsp;</div> <div> &nbsp;</div> <div> $\angle KAE=2\angle KDE$ y $\angle KDE=\angle CAE$, entonces $AC$ es todotriz de $\angle EAK$.&nbsp;</div> <div> &nbsp;</div> <div> Con esto concluimos que $K$ es el ortocentro del tri&aacute;ngulo $ABC$.</div> </div> </div> </div> </fieldset> https://www.matetam.com/problemas/geometria/p5-dos-circunferencias-una-perpendicular#comments Geometría Avanzado Selectivo Final OMM Tamaulipas 2024 Sat, 19 Oct 2024 20:12:48 +0000 Samuel Elias 4132 at https://www.matetam.com P4. Ceros y Unos en un pizarrón. https://www.matetam.com/problemas/combinatoria/p4-ceros-y-unos-un-pizarron <div> Sea $n$ entero positivo. Hay $2n$ n&uacute;meros escritos en el pizarr&oacute;n: $n$ 0&rsquo;s y $n$ 1&rsquo;s. Una movida consiste en escoger dos n&uacute;meros del pizarr&oacute;n, borrarlos y escribir 0 si eran iguales o 1 si eran distintos. Despues de hacer varias movidas, queda solo un n&uacute;mero.</div> <ul> <li> <span style="font-size: 1.2rem;">&iquest;Para qu&eacute; valores de $n$ te puede quedar un n&uacute;mero par?</span></li> <li> &iquest;Para qu&eacute; valores de $n$ te puede quedar un n&uacute;mero impar?</li> </ul> <div> &nbsp; &nbsp;&nbsp;</div> <fieldset class="fieldgroup group-sugerencia"><legend>Sugerencia</legend><div class="field field-type-text field-field-sugerencia"> <div class="field-label">Sugerencia:&nbsp;</div> <div class="field-items"> <div class="field-item odd"> <p>Invarianza? Observa que la paridad de la suma no cambia. &iquest;C&oacute;mo se comporta un movimiento de forma individual?</p> </div> </div> </div> </fieldset> <fieldset class="fieldgroup group-sol-sep"><legend>Solución</legend><div class="field field-type-text field-field-sol"> <div class="field-label">Solución:&nbsp;</div> <div class="field-items"> <div class="field-item odd"> <div> Vamos a usar el siguiente lema</div> <div> &nbsp;</div> <div> <strong>Lema 1</strong>: La paridad de la suma de los n&uacute;meros en el pizarr&oacute;n nunca cambia.</div> <div> &nbsp;</div> <div> Notemos que si eliminamos dos n&uacute;meros iguales (digamos $a$) y agregamos un 0, entonces la suma disminuy&oacute; por $2a$ y aument&oacute; por 0. Entonces la suma cambi&oacute; por un n&uacute;mero par por lo que su paridad no cambia. Si eliminamos dos n&uacute;meros diferentes, entonces tiene que ser un 0 y un 1, y agregamos un 1, entonces la suma disminuy&oacute; en 1 y aument&oacute; en 1 por lo que permaneci&oacute; igual. Entonces la paridad no cambia.</div> <div> &nbsp;</div> <div> C&oacute;mo los n&uacute;meros en el pizarr&oacute;n eran originalmente 0&#39;s y 1&#39;s, y cada vez que agregamos un n&uacute;mero es un 0 o un 1, entonces &uacute;nicamente solo habr&aacute;n 0&#39;s y 1&#39;s en el pizarr&oacute;n.</div> <div> &nbsp;</div> <div> Si $n$ es impar entonces la suma original del pizarr&oacute;n es $n$ que es impar por lo que el n&uacute;mero final tendr&aacute; que ser impar para preservar la paridad por lo que ser&aacute; un 1. Si $n$ es par entonces la suma original es $n$ que es par lo que lo el n&uacute;mero final tendr&aacute; que ser 0 para preservar la paridad.&nbsp;</div> <div> &nbsp;</div> </div> </div> </div> </fieldset> https://www.matetam.com/problemas/combinatoria/p4-ceros-y-unos-un-pizarron#comments Combinatoria Intermedio Selectivo Final OMM Tamaulipas 2024 Sat, 19 Oct 2024 20:08:46 +0000 Samuel Elias 4131 at https://www.matetam.com P3. Desigualdades en un selectivo https://www.matetam.com/problemas/algebra/p3-desigualdades-un-selectivo <p>Sean $a,b,c$ n&uacute;meros reales positivos tales que $abc=\frac{1}{8}$. Demuestra que: \[a^2+b^2+c^2+a^2b^2+a^2c^2+b^2c^2\geq\frac{15}{16}\]</p> <fieldset class="fieldgroup group-sugerencia"><legend>Sugerencia</legend><div class="field field-type-text field-field-sugerencia"> <div class="field-label">Sugerencia:&nbsp;</div> <div class="field-items"> <div class="field-item odd"> <p>Estudia $a^2+b^2+c^2$ y $a^2b^2+a^2c^2+b^2c^2$ por separado. Puedes usar $MA-MG$ o alguna otra desigualdad que conozcas.&nbsp;</p> </div> </div> </div> </fieldset> <fieldset class="fieldgroup group-sol-sep"><legend>Solución</legend><div class="field field-type-text field-field-sol"> <div class="field-label">Solución:&nbsp;</div> <div class="field-items"> <div class="field-item odd"> <div> Por $MA-MG$, $$a^2+b^2+c^2 \geq 3\sqrt[3]{a^2b^2c^2}=3\left(\sqrt[3]{\frac{1}{8}}\right)^2=3 \cdot \frac{1}{4}=\frac{3}{4}$$</div> <div> Tambi&eacute;n por $MA-MG$,</div> <div> $$a^2b^2+a^2c^2+b^2c^2\geq 3\sqrt[3]{a^2a^2b^2b^2c^2c^2}=3\sqrt[3]{(abc)^4}=3\sqrt[3]{\frac{1}{8^4}}=\frac{3}{16}$$</div> <div> Sumando ambas desigualdades, $$a^2+b^2+c^2+a^2c^2+a^2b^2+b^2c^2 \geq \frac{3}{4}+\frac{3}{16}=\frac{12+3}{16}=\frac{15}{16}$$</div> </div> </div> </div> </fieldset> https://www.matetam.com/problemas/algebra/p3-desigualdades-un-selectivo#comments Álgebra Avanzado Selectivo Final OMM Tamaulipas 2024 Sat, 19 Oct 2024 20:05:42 +0000 Samuel Elias 4130 at https://www.matetam.com P2. Los monos de Daniel https://www.matetam.com/problemas/combinatoria/p2-los-monos-daniel <p>Daniel tiene 1600 pl&aacute;tanos y 100 monos. &Eacute;l va a repartir sus pl&aacute;tanos entre sus 100 monos (pero no de forma justa, algunos tendr&aacute;n m&aacute;s pl&aacute;tanos que otros, incluso habr&aacute; monos que no reciban ning&uacute;n pl&aacute;tano). Demuestra que al menos 4 monos tendr&aacute;n la misma cantidad de pl&aacute;tanos.</p> <fieldset class="fieldgroup group-sugerencia"><legend>Sugerencia</legend><div class="field field-type-text field-field-sugerencia"> <div class="field-label">Sugerencia:&nbsp;</div> <div class="field-items"> <div class="field-item odd"> <p>&iquest;C&oacute;mo deber&aacute; repartir Daniel sus pl&aacute;tanos para que 4 monos no repitan la misma cantidad?</p> </div> </div> </div> </fieldset> <fieldset class="fieldgroup group-sol-sep"><legend>Solución</legend><div class="field field-type-text field-field-sol"> <div class="field-label">Solución:&nbsp;</div> <div class="field-items"> <div class="field-item odd"> <div> Por principio de casillas, el acomodo para evitar que 4 monos tengan el mismo n&uacute;mero de pl&aacute;tanos es:</div> <div> &nbsp;</div> <div> <span style="font-size: 1.2rem;">&nbsp;A 3 monos no le damos pl&aacute;tano</span></div> <div> &nbsp;A 3 monos le damos uno</div> <div> &nbsp;A 3 monos le damos dos</div> <div> &nbsp;</div> <div> &nbsp;</div> <div> Y as&iacute; sucesivamente. Entonces podemos hacer en total 33 grupos de 3 y un mono queda solito, que para evitar que tenga la misma cantidad de pl&aacute;tanos que los dem&aacute;s, le tenemos que dar 33. Pero observe, que la cantidad de pl&aacute;tanos que necesitamos para poder hacer esto va a ser 3 veces la suma de Gauss del 1 al 32 mas los 33 que ocupamos, que es:</div> <div> $$3\cdot \frac{(32)(33)}{2}+33=1617$$</div> <div> Entonces nos faltan 17 pl&aacute;tanos para que esto sea posible.&nbsp;</div> </div> </div> </div> </fieldset> https://www.matetam.com/problemas/combinatoria/p2-los-monos-daniel#comments Combinatoria Intermedio Selectivo Final OMM Tamaulipas 2024 Sat, 19 Oct 2024 20:02:48 +0000 Samuel Elias 4129 at https://www.matetam.com P1. Repaso de la cantidad de divisores de un número. https://www.matetam.com/problemas/numeros/p1-repaso-cantidad-divisores-un-numero <div> Un entero positivo $n$ tiene exactamente 2 divisores, mientras que el n&uacute;mero $n + 1$ tiene exactamente 3</div> <div> divisores. &iquest;Cu&aacute;l es la mayor cantidad de divisores que puede tener el n&uacute;mero $n + 2$?</div> <fieldset class="fieldgroup group-sugerencia"><legend>Sugerencia</legend><div class="field field-type-text field-field-sugerencia"> <div class="field-label">Sugerencia:&nbsp;</div> <div class="field-items"> <div class="field-item odd"> <p>Observa que $n$ es primo. Cu&aacute;l es el &uacute;nico $n+1$ despu&eacute;s de un primo con 3 divisores?</p> </div> </div> </div> </fieldset> <fieldset class="fieldgroup group-sol-sep"><legend>Solución</legend><div class="field field-type-text field-field-sol"> <div class="field-label">Solución:&nbsp;</div> <div class="field-items"> <div class="field-item odd"> <p>Sea $d(n)$ la cantidad de divisores de $n$. Como $d(n)=2$, es un hecho conocido que $d(p)=2$ siendo $p$ un n&uacute;mero primo, entonces $n$ es primo. A su vez, es un hecho conocido que $d(p^2)=3$. Entonces, $n+1=p^2 \iff n=(p+1)(p-1)$, y como $n$ es un entero positivo, $p+1=n, \ p-1=1 \iff p=2 \iff n=3$. $\therefore n+2=5 \Rightarrow d(5)=2$.</p> </div> </div> </div> </fieldset> https://www.matetam.com/problemas/numeros/p1-repaso-cantidad-divisores-un-numero#comments Números Básico Selectivo Final OMM Tamaulipas 2024 Sat, 19 Oct 2024 20:00:25 +0000 Samuel Elias 4128 at https://www.matetam.com 3.- Los delegados de Tamaulipas jugando una modificación de ajedrez https://www.matetam.com/problemas/combinatoria/3-los-delegados-tamaulipas-jugando-una-modificacion-ajedrez <p>Considera un tablero de ajedrez de $8 \times 8$. Orlando y Mois&eacute;s juegan alternando turnos, comenzando por Orlando. Cada uno en su turno coloca un alfil en alguna casilla del tablero vac&iacute;a, de tal forma que los alfiles no se ataquen entre s&iacute;. Pierde el jugador que coloque un alfil que sea atacado por otro previamente. Si los alfiles son del mismo color (es decir, o tienen puros alfiles blancos o puros alfiles negros), determina qui&eacute;n tiene una estrategia ganadora y descr&iacute;bela.&nbsp;<br /> <strong>Nota:&nbsp;</strong>un jugador puede atacarse a s&iacute; mismo.&nbsp;</p> <fieldset class="fieldgroup group-sugerencia"><legend>Sugerencia</legend><div class="field field-type-text field-field-sugerencia"> <div class="field-label">Sugerencia:&nbsp;</div> <div class="field-items"> <div class="field-item odd"> <p>Simetr&iacute;a de espejo?? Demuestra que si partes el tablero de ajedrez a la mitad, Mois&eacute;s podr&aacute; copiar los movimientos de Orlando y eventualmente ganar.&nbsp;</p> </div> </div> </div> </fieldset> <fieldset class="fieldgroup group-sol-sep"><legend>Solución</legend><div class="field field-type-text field-field-sol"> <div class="field-label">Solución:&nbsp;</div> <div class="field-items"> <div class="field-item odd"> <p>La estrategia se basa en simetr&iacute;a: Si Orlando puso un alfil en la casilla $(i,j)$ entonces Mois&eacute;s puede poner un alfil en la casilla $(9-i,j)$. Si Mois&eacute;s no puede poner un alfil en esa casilla, significa que esa casilla ya andaba atacada por otro alfil en la misma diagonal, digamos que hab&iacute;a un alfil en la casilla $(9-i+\epsilon_1\cdot k,j+\epsilon_2\cdot k)$ donde $\epsilon_1$ y $\epsilon_2$ son $\pm 1$. Pero por la estrategia de simetr&iacute;a, eso signfica que si Orlando puso una ficha ah&iacute;, entonces Mois&eacute;s debi&oacute; poner una ficha en $(9-(9-i+\epsilon_1\cdot k),j+\epsilon_2\cdot k)=(i-\epsilon_1\cdot k,j+\epsilon_2\cdot k)$ lo cual implica que la casilla $(i,j)$ ya andaba atacada y entonces Orlando nunca pudo poner una ficha ah&iacute;. De manera an&aacute;loga si Mois&eacute;s fue el que puso la ficha en $(9-i+\epsilon_1\cdot k,j+\epsilon_2\cdot k)$ entonces Orlando debi&oacute; poner antes un alfil en $(i-\epsilon_1\cdot k,j+\epsilon_2\cdot k)$ lo cual llega a la misma contradicci&oacute;n. Como el tablero es de $8 \times 8$, eventualmente Orlando se va a quedar sin movimientos.&nbsp;</p> </div> </div> </div> </fieldset> https://www.matetam.com/problemas/combinatoria/3-los-delegados-tamaulipas-jugando-una-modificacion-ajedrez#comments Combinatoria Intermedio Semifinal OMM Tamaulipas 2024 Sat, 19 Oct 2024 19:57:30 +0000 Samuel Elias 4127 at https://www.matetam.com 2.- Ecuación de ternas en progresión Geométrica https://www.matetam.com/problemas/algebra/2-ecuacion-ternas-progresion-geometrica <p>Determina todas las ternas de n&uacute;meros naturales $(a,b,c)$ con $0&lt;a&lt;b&lt;c$ en progresi&oacute;n geom&eacute;trica para las cuales se cumplen las siguientes dos ecuaciones:&nbsp;</p> <p>$$a+b+c=35$$</p> <p>$$a^2+b^2+c^2=525$$</p> <fieldset class="fieldgroup group-sugerencia"><legend>Sugerencia</legend><div class="field field-type-text field-field-sugerencia"> <div class="field-label">Sugerencia:&nbsp;</div> <div class="field-items"> <div class="field-item odd"> <p>Si tienen una raz&oacute;n $r$ tal que $b=ar$ y $c=br$, &iquest;cu&aacute;nto vale c en t&eacute;rminos de $a$?<br /> <br /> Despu&eacute;s de sustituir, usa divisibilidad porque $a$ es entero y checa todos los casos.&nbsp;</p> </div> </div> </div> </fieldset> <fieldset class="fieldgroup group-sol-sep"><legend>Solución</legend><div class="field field-type-text field-field-sol"> <div class="field-label">Solución:&nbsp;</div> <div class="field-items"> <div class="field-item odd"> <div> Sea $r$ la raz&oacute;n de la progresi&oacute;n geom&eacute;trica. Entonces, digamos que $a=a$, $b=ar$ y $c=ar^2$. Como $a&lt;b&lt;c$, $r&gt;1$. Ahora, observe que $a+ar+ar^2=35 \iff a(r^2+r+1)=35$. Como $a \in \ \mathbb{N}$, entonces $a=1, 5, 7, 35$.&nbsp;</div> <div> &nbsp;</div> <div> &nbsp; &nbsp; Caso 1. $a=1$</div> <div> &nbsp;</div> <div> Entonces nos queda que $r^2+r-34=0$, entonces usando chicharronera $$r=\frac{-1\pm \sqrt{1^2-(4)(1)(-34)}}{2(1)} \iff r=\frac{-1\pm \sqrt{137}}{2}$$ Pero esto haria que $b$ no sea natural</div> <div> Caso 2. $a=5$, entonces $r^2+r+1=7 \iff r^2+r-6=0 \iff (r-2)(r+3)=0$, y como $r&gt;1$, $r=2$. Esto nos da la terna $(5, 10, 20)$. Confirmando en la segunda ecuaci&oacute;n: $$25+100+400=525$$</div> <div> &nbsp;</div> <div> &nbsp;Caso 3. $a=7$. Entonces $r^2+r+1=5 \iff r^2+r-4=0$. Usando chicharronera se llega a que $r=\frac{-1\pm \sqrt{17}}{2}$, lo cual haria que $b$ no sea natural</div> <div> &nbsp;</div> <div> Caso 4. $a=35$. Entonces $r^2+r+1=1 \iff r^2+r=0 \iff r(r+1)=0$, pero $r&gt;1.$&nbsp;</div> <div> &nbsp;</div> <div> &nbsp;</div> <div> La &uacute;nica terna que cumple es la $(5,10,20)$</div> </div> </div> </div> </fieldset> https://www.matetam.com/problemas/algebra/2-ecuacion-ternas-progresion-geometrica#comments Álgebra Números Básico Semifinal OMM Tamaulipas 2024 Sat, 19 Oct 2024 19:47:20 +0000 Samuel Elias 4126 at https://www.matetam.com 1.- Aprovecha el radio con isósceles. https://www.matetam.com/problemas/geometria/1-aprovecha-radio-isosceles <p>Sea $ABC$ un tri&aacute;ngulo tal que $ABC=60$&deg; y sea $O$ su circuncentro de tal forma que $CBO=45$&deg;. La recta $BO$ corta al segmento $AC$ en $D$. Demuestra que el tri&aacute;ngulo AOD es is&oacute;sceles y encuentra la medida de sus &aacute;ngulos.&nbsp;&nbsp;</p> <fieldset class="fieldgroup group-sugerencia"><legend>Sugerencia</legend><div class="field field-type-text field-field-sugerencia"> <div class="field-label">Sugerencia:&nbsp;</div> <div class="field-items"> <div class="field-item odd"> <p>Recuerda que el circunentro te genera muchos tri&aacute;ngulos is&oacute;sceles&nbsp;</p> </div> </div> </div> </fieldset> <fieldset class="fieldgroup group-sol-sep"><legend>Solución</legend><div class="field field-type-text field-field-sol"> <div class="field-label">Solución:&nbsp;</div> <div class="field-items"> <div class="field-item odd"> <div> Observa que como $\angle CBO=45$, entonces $\angle ABO=60-45=15$. Como tenemos is&oacute;sceles formados por los radios, entonces $\angle BAO=15$, y por angulo exterior de un tri&aacute;ngulo, $\angle AOD=30$.&nbsp;</div> <div> &nbsp;</div> <div> Ahora, como $\angle CBO=45$, $ \angle BCO=45$ y por exterior del tri&aacute;ngulo, entonces $\angle DOC=90$.&nbsp;</div> <div> &nbsp;</div> <div> Por &uacute;ltimo, como $\angle AOC=120$ y $\triangle AOC$ es is&oacute;sceles, entonces $\angle CAO=\angle ACO=30$, entonces como $\angle DOA = \angle DAO$, el $\triangle AOD$ es is&oacute;sceles y sus &aacute;ngulos son 30, 30, 120 (en ese orden de v&eacute;rtices).</div> </div> </div> </div> </fieldset> https://www.matetam.com/problemas/geometria/1-aprovecha-radio-isosceles#comments Geometría Básico Semifinal OMM Tamaulipas 2024 Sat, 19 Oct 2024 19:40:26 +0000 Samuel Elias 4125 at https://www.matetam.com Comentarios del Estatal de 2024 https://www.matetam.com/blog/entradas-samuel-elias/comentarios-del-estatal-2024 <a href="/blog/entradas-samuel-elias/comentarios-del-estatal-2024"></a><a href="/blog/entradas-samuel-elias/comentarios-del-estatal-2024"></a><p>Particularmente este a&ntilde;o, he sentido algo complicado el proceso selectivo. Se nos han juntado muchas fechas, hemos tenido que revisar a altas horas de la noche y conseguir apoyo a terceros es cada vez m&aacute;s complicado.&nbsp;</p> <p>Sobre el examen, dar&eacute; mis comentarios por problema (los problemas ya se publicaron en la secci&oacute;n de problemas, ver Estatal 2024).&nbsp;</p><p><a href="https://www.matetam.com/blog/entradas-samuel-elias/comentarios-del-estatal-2024" target="_blank">leer más</a></p> https://www.matetam.com/blog/entradas-samuel-elias/comentarios-del-estatal-2024#comments XXXVIII OMM 2024 Sun, 29 Sep 2024 18:25:21 +0000 Samuel Elias 4124 at https://www.matetam.com